PT28.S3.Q12 - letter to the editor: your article on effective cockroach control

Samcandoit!Samcandoit! Alum Member
edited January 2016 in Logical Reasoning 71 karma
http://7sage.com/lsat_explanations/lsat-28-section-3-question-12/
I really do not understand why the answer is C, and the question stem also seems to be very confusing. Are we suppose to find one option that is "must be false", or there are four "must be false" answers, we need to choose the one that is not? The first time, I treat this as a normal must be false question, thought there is only one must be false choice. I chose A. Because the stem said that V is effective to all the species that infest NA; while A said it is only effective to two species.
Someone help me plz.

Comments

  • Matt1234567Matt1234567 Inactive ⭐
    1294 karma
    The question stem says "each of the following statements CONFLICT with the writers view, EXCEPT"

    Four answer choices will conflict, and one has either no bearing on the facts or will be aligned with the facts.

    A) this conflicts with the stimulus because the stimulus states that vexon has been utilized effectively against ALL of the species that infest NA.
    B) this conflicts with the stimulus because the stimulus states that Roach Ender contains vexon, and vexon is effective against all species that infest NA.
    C) This is the correct answer choice because it is essentially a reinstatement of the facts from the stimulus. We know that vexon is effective against all species that infest North America, and this answer says the exact same thing.
    D) Conflicts with the stimulus, its effective against all species.
    E) Conflicts with the stimulus, more than 4,000 species were tested.
Sign In or Register to comment.